ECON Chap. 4: Market Failures; Chap. 6: Elasticity; Chap. 9 Costs of Production.

Lakukan tugas rumah & ujian kamu dengan baik sekarang menggunakan Quizwiz!

Answer the question on the basis of the following cost data. Output Total Cost 0 $24 1 $33 2 $41 3 $48 4 $54 5 $61 6 $69 The average fixed cost of producing 3 units of output is Multiple Choice A) $6. B) $7.40. C) $8. D) $5.50.

$8. Correct

When the price of a product is increases by 15 percent, the quantity demanded decreases by 10 percent. We can therefore conclude that the demand for this product is Multiple Choice A. inelastic. B. elastic. C. unitary elastic. D. cross-elastic.

A. inelastic. Correct

Ouput Total Cost 0 $10 1 $20 2 $28 3 $38 4 $53 5 $73 6 $98 Refer to the provided table. The marginal cost of producing the sixth unit of output is Multiple Choice A) $98. B) $25. C) $10. D) $16.33.

B) $25. Correct

Refer to the provided graph. Minimum efficient scale occurs at Multiple Choice A) Q3. B) Q2. C) Q1. D) Q4.

B) Q2. Correct

The fixed cost of the firm is $500. The firm's total variable cost is indicated in the table. output Total Variable Cost 1 $400 2 $720 3 $1,000 4 $1,400 5 $2,000 6 $3,600 The average total cost of the firm when 3 units of output are being produced is Multiple Choice A) $500. B) $350. C) $700. D) $400.

A) $500. Correct TC= FC + VC TC= $1,000 + $500 = $1,500 ATC= TC/Q ATC = $1,500/3 = $500

In a free-market economy, a product which entails a positive externality will be Multiple Choice A. underproduced. B. overproduced. C. produced at the optimal level. D. provided solely by the government.

A. underproduced. Correct

Economic profits are calculated by subtracting Multiple Choice A. implicit costs from total revenue. B. explicit and implicit costs from total revenue. C. implicit costs from normal profits. D. explicit costs from total revenue.

B. explicit and implicit costs from total revenue. Correct

Refer to the graph. Diminishing marginal returns are reflected in Multiple Choice A) a move along short-run average total cost curve ATC1 from point b to point a. B) the shift of the short-run average total cost curve from ATC1 to ATC2. C) a move along short-run average total cost curve ATC2 from point e to point f. Correct D) the shift of the short-run average total cost curve from ATC2 to ATC1.

C) a move along short-run average total cost curve ATC2 from point e to point f. Correct

The larger the diameter of a natural gas pipeline, the lower is the average total cost of transmitting 1,000 cubic feet of gas 1,000 miles. This is an example of one reason for Multiple Choice A) diminishing marginal returns. B) economies of scale. C) diminishing returns to scale. D) increasing marginal cost.

C) economies of scale. Correct

When a firm does more of something, it gets better at it. This learning-by-doing is Multiple Choice A) a source of diseconomies of scale. B) called the principle of natural progression. C) called "spreading the overhead." D) a source of economies of scale.

D) a source of economies of scale. Correct

If quantity demanded is completely unresponsive to price changes, demand is Multiple Choice A. relatively elastic. B. relatively inelastic. C. perfectly elastic. D. perfectly inelastic.

D. perfectly inelastic. Correct

A normal good would have a positive price-elasticity of demand. True or False

False

A positive cross-elasticity of demand between two goods indicates that the two goods are both normal goods. True or False

False

Along a supply curve, product price and producer surplus are inversely related. True or False

False

Excludability means that when someone is consuming a good, then others are excluded from using the good anymore. True or False

False

The government receives all of the benefits associated with the production of a public good. True or False

False

The optimal quantity of a public good is where the total benefits from it are equal to the total costs of producing it. True or False

False

The smaller the number of good substitutes for a product, the greater will be the price elasticity of demand for it. True or False

False

Allocative efficiency occurs where the collective sum of consumer and producer surplus is at a maximum. True or False

True

Along a demand curve, product price and consumer surplus are inversely related. True or False

True

eBay and Amazon provide "sellers' ratings" information based on the experiences of past buyers. This is to help resolve the adverse selection problem faced by potential buyers. True or False

True

In the provided graph, the equilibrium point in the market is where the S and D curves intersect. At equilibrium, the total revenues received by sellers would be represented by the area Multiple Choice A. b + c + d. B. a + b. C. b. D. b + c.

D. b + c. Correct

A cross elasticity of demand coefficient of +2.5 indicates that the two products are substitutes. True or False

True

Answer the question on the basis of the following information. Number of Workers Total Product Marginal Product 0 0 --- 1 8 8 2 10 3 25 4 30 5 3 6 34 The marginal product of the fourth worker Multiple Choice A) is 7 1/2. B) is 7. C) is 5. D) cannot be calculated from the information given.

is 5. Correct

If the coefficient of income elasticity of demand is positive, the product is an inferior good. True or False

False

If the price of labor or some other variable resource decreased, the Multiple Choice A) MC curve would shift downward. B) AVC curve would shift upward. C) AFC curve would shift upward. D) AFC curve would shift downward.

A) MC curve would shift downward. Correct

In the short run, total output in an industry Multiple Choice A) can vary as the result of using a fixed amount of plant and equipment more or less intensively. B) may be altered by varying the size of plant and equipment which now exist in the industry. C) is fixed at a specific level. D) can vary as the result of new firms entering or leaving the industry.

A) can vary as the result of using a fixed amount of plant and equipment more or less intensively. Correct

If in the short run a firm's total product is increasing, then its Multiple Choice A) marginal product could be either increasing or decreasing. B) marginal product must be decreasing. C) marginal product must also be increasing. D) average product must also be increasing.

A) marginal product could be either increasing or decreasing. Correct

Refer to the provided graph. At which point does marginal product (MP) equal average product (AP) at a specific level of output? Multiple Choice A) point B B) point D C) point A D) point C

A) point B Correct

The mass affordability of the iPhone is the result of the following, except Multiple Choice A) the law of diminishing returns in manufacturing. B) mass production and spreading of fixed costs. C) economies of scale and large volumes. D) mass sales and distribution cost savings.

A) the law of diminishing returns in manufacturing. Correct

The question is based on the following table, which provides information on the production of a product that requires one variable input. Input Total Product 0 0 1 5 2 20 3 32 4 42 5 50 6 55 7 58 8 58 9 56 Diminishing marginal returns sets in with the addition of the Multiple Choice A) third unit of input. B) fourth unit of input. C) second unit of input. D) first unit of input.

A) third unit of input. Correct

Charlie is willing to pay $10 for a T-shirt that is priced at $9. If Charlie buys the T-shirt, then his consumer surplus is Multiple Choice A. $1. B. $0.90. C. $90. D. $19.

A. $1. Correct

Answer the question based on the following data. Price Per Unit Quantity Demanded Per Unit of Time $20 12 $18 17 $16 20 $14 24 $12 30 $10 36 $8 40 $6 44 $4 48 Over which of the following price ranges is the demand unit-elastic? Multiple Choice A. $12-$10 B. $16-$14 C. $18-$16 D. $14-$12

A. $12-$10 Correct

Product Minimum Acceptable Price Actual Price A $6 $13 B $7 $13 C $9 $13 D $11 $13 Refer to the provided table. What is the producer surplus for all producers A, B, C, and D? Multiple Choice A. $19 B. $6 C. $24 D. $13

A. $19 Correct

Harvey quit his job at State University, where he earned $45,000 a year. He figures his entrepreneurial talent or forgone entrepreneurial income to be $5,000 a year. To start the business, he cashed in $100,000 in bonds that earned 10 percent interest annually to buy a software company, Extreme Gaming. In the first year, the firm sold 11,000 units of software at $75 for each unit. Of the $75 per unit, $55 goes for the costs of production, packaging, marketing, employee wages and benefits, and rent on a building. The total revenues of Harvey's firm in the first year were Multiple Choice A. $825,000. B. $105,000. C. $220,000. D. $605,000.

A. $825,000. Correct Revenue = 11,000 * 75 = $825,000 Revenue = # of units * price per unit

When the price of movie tickets in a certain town was reduced, the movietheaters' revenues did not change. This suggests that the demand for movie tickets in that town has a price-elasticity coefficient of Multiple Choice A. 1.0. B. 0.5. C. zero. D. greater than 1.

A. 1.0. Correct

Suppose the price of local cable TV service increased from $16.20 to $19.80 and as a result the number of cable subscribers decreased from 224,000 to 176,000. Along this portion of the demand curve, price elasticity of demand is Multiple Choice A. 1.2. B. 1.6. C. 0.8. D. 8.0.

A. 1.2. Correct

Depositors do not check their banks carefully for stability anymore, because of the Federal deposit insurance program. This illustrates the problem of Multiple Choice A. public goods. B. adverse selection. C. moral hazard. D. externalities.

C. moral hazard. Correct

Answer the question on the basis of the following information for a public good. Pa and Pb are the prices that individuals A and B are willing to pay for the last unit of a public good, rather than do without it. These people are the only two members of society. Q Pa Pb 1 $3 $5 2 2 4 3 1 3 4 0 2 5 0 1 If the marginal cost of producing this good at the optimal quantity is $4, the optimal quantity must be Multiple Choice A. 3 units. B. 2 units. C. 4 units. D. 1 unit.

A. 3 units. Correct

Answer the question on the basis of the following information for a public good. Pa and Pb are the prices that individuals A and B are willing to pay for the last unit of a public good, rather than do without it. These people are the only two members of society. Q Pa Pb 1 $3 $5 2 2 4 3 1 3 4 0 2 5 0 1 If this good were a private good instead of a public one, the total quantity demanded at a $3 market price would be Multiple Choice A. 4 units. B. 3 units. C. 2 units. D. 6 units.

A. 4 units. Correct

Refer to the diagram. From society's perspective, if MB2 and MC1 are relevant, Multiple Choice A. Q1 represents too little pollution abatement. Correct B. Q4 represents too little pollution abatement. C. Q2 represents too little pollution abatement. D. Q3 represents an optimal amount of pollution abatement.

A. Q1 represents too little pollution abatement. Correct

Refer to the above graph. Which of the following statements is correct? Multiple Choice A. Supply is perfectly elastic. B. Demand is perfectly inelastic. C. Supply is perfectly inelastic. D. Demand is perfectly elastic.

A. Supply is perfectly elastic. Correct

Graphically, producer surplus is measured as the area Multiple Choice A. above the supply curve and below the actual price. Correct B. under the demand curve and below the actual price. C. under the demand curve and above the actual price. D. above the supply curve and above the actual price.

A. above the supply curve and below the actual price. Correct

Suppose a firm offers its workers a cafeteria plan in which it allows workers to allocate a set amount of fringe benefit money toward specific insurance. Mary, who has five kids needing braces, selects the family dental coverage. This is an example of the Multiple Choice A. adverse selection problem. B. free-rider problem. C. principal-agent problem. D. moral hazard problem.

A. adverse selection problem. Correct

Refer to the diagram of the market for product X. Curve St embodies all costs (including externalities), and Dt embodies all benefits (including externalities) associated with the production and consumption of X. Assuming the market equilibrium output is Q1, we can conclude that the existence of external Multiple Choice A. benefits has resulted in an underallocation of resources to X. B. costs has resulted in an underallocation of resources to X. C. benefits has resulted in an overallocation of resources to X. D. costs has resulted in an overallocation of resources to X.

A. benefits has resulted in an underallocation of resources to X. Correct

It has been proposed that a government agency be charged with the task of determining the amount of pollution that the atmosphere (or a body of water) can safely absorb, establish "rights" to this limited amount of pollution, and sell those limited amount of rights to firms. The firms can then buy and sell these rights among themselves later. This approach is known as the Multiple Choice A. cap-and-trade system. B. taxes and subsidies system. C. market and command system. D. property rights system.

A. cap-and-trade system. Correct

The MB curves in the diagram slope downward because of the law of Multiple Choice A. diminishing marginal utility. B. diminishing returns. C. conservation of matter and energy. D. increasing cost.

A. diminishing marginal utility. Correct

Refer to the provided supply and demand graph for a product. In the graph, line S is the current supply of this product, while line S1 is the optimal supply from the society's perspective. This figure suggests that there is (are) Multiple Choice A. external costs in the production of this product. B. positive externalities from producing the product. C. external benefits from the production of this product. D. currently an underallocation of resources toward producing this product.

A. external costs in the production of this product. Correct

You are the only seller of eggs in town, and the price-elasticity coefficient for eggs is known to be 0.8. If you want to increase your sales quantity by 10 percent through a price change, what should you do to price? Multiple Choice A. increase price by 12.5 percent B. increase price by 8 percent C. reduce price by 12.5 percent D. reduce price by 8 percent

A. increase price by 12.5 percent.

If the demand for product X is inelastic, a 4 percent decrease in the price of X will Multiple Choice A. increase the quantity of X demanded by less than 4 percent. B. decrease the quantity of X demanded by more than 4 percent. C. decrease the quantity of X demanded by less than 4 percent. D. increase the quantity of X demanded by more than 4 percent.

A. increase the quantity of X demanded by less than 4 percent.

The income elasticity of demand for jewelry is +2. Other things equal, a 10 percent increase in consumer income will Multiple Choice A. increase the quantity of jewelry purchased by 20 percent. Correct B. decrease the quantity of jewelry purchased by 5 percent. C. increase the quantity of jewelry purchased by 5 percent. D. decrease the quantity of jewelry purchased by 20 percent.

A. increase the quantity of jewelry purchased by 20 percent. Correct

Price Quantity Supplied $10 10 $8 9 $6 8 $4 7 $2 6 Refer to the table. Over the $8-$6 price range, supply is Multiple Choice A. inelastic. B. perfectly inelastic. C. elastic. D. perfectly elastic.

A. inelastic. Correct

Elasticity of supply will increase when Multiple Choice A. it becomes easier to substitute one factor of production for another in a manufacturing process. B. the number of producers selling a product decreases. C. the number of consumers wanting to purchase a product increases. D. producers are given less time to respond to price changes.

A. it becomes easier to substitute one factor of production for another in a manufacturing process. Correct

Refer to the diagram. Which one of the following might shift the marginal benefit curve from MB1 to MB2? Multiple Choice A. major new studies strongly linking cancer to pollution. B. improved technology for reducing pollution. C. a decrease in the price of recycled goods. D. a change in consumer tastes from manufactured goods to services.

A. major new studies strongly linking cancer to pollution. Correct

Suppose that a 10 percent increase in the price of normal good Y causes a 20 percent increase in the quantity demanded of normal good X. The coefficient of cross elasticity of demand is Multiple Choice A. positive, and therefore these goods are substitutes. B. positive, and therefore these goods are complements. C. negative, and therefore these goods are complements. D. negative, and therefore these goods are substitutes.

A. positive, and therefore these goods are substitutes. Correct

Refer to the provided supply and demand graph of Product X. What would happen if the government decided to also start providing Product X in the market? Multiple Choice A. price would decrease B. demand would decrease C. demand would increase D. supply would decrease

A. price would decrease Correct

The market system does not produce public goods because Multiple Choice A. private firms cannot stop consumers who are unwilling to pay for such goods from benefiting from them. B. their production seriously distorts the distribution of income. C. there is no need or demand for such goods. D. public enterprises can produce such goods at lower cost than can private enterprises.

A. private firms cannot stop consumers who are unwilling to pay for such goods from benefiting from them. Correct

Which statement is correct? Multiple Choice A) If average variable cost is increasing, then average total cost must be increasing too. B) The marginal cost curve cuts the average variable cost curve at its lowest point. C) The marginal cost curve cuts the average variable cost curve at an output greater than where the marginal cost curve cuts the average cost curve. D) Marginal cost is the change in average cost when there is a change in output of 1 unit.

B) The marginal cost curve cuts the average variable cost curve at its lowest point. Correct

If an industry's long-run average total cost curve has an extended range of constant returns to scale, this implies that Multiple Choice A) the industry will comprise a very large number of small firms. B) both relatively small and relatively large firms can be viable in the industry. C) technology precludes both economies and diseconomies of scale. D) the industry will be a natural monopoly.

B) both relatively small and relatively large firms can be viable in the industry. Correct

As output increases, average fixed costs Multiple Choice A) first increase and then decrease. B) decrease. C) remain constant. D) increase.

B) decrease. Correct

Over the range of positive, but diminishing, marginal returns for an input, the total product curve Multiple Choice A) falls. B) rises at a decreasing rate. C) rises at an increasing rate. D) rises at a constant rate.

B) rises at a decreasing rate. Correct

Suppose that a business incurred implicit costs of $200,000 and explicit costs of $1 million in a specific year. If the firm sold 4,000 units of its output at $300 per unit, its accounting profits were Multiple Choice A. $0 and its economic loss was $200,000. B. $200,000 and its economic profits were $0. C. $100,000 and its economic profits were $100,000. D. $100,000 and its economic profits were $0.

B. $200,000 and its economic profits were $0. Correct 1. Total Revenue = 4,000 * $300 = $1,200,000 2. Accounting Profit = Revenue - explicit cost Accounting Profit = $1,200,000 - $1,000,000 = $200,000 3. Economic Profit = Revenue - All costs (explicit + implicit) Economic Profit = $1,200,000 - ($1,000,000 + $200,000) = $0

If average variable cost is $74 and total fixed cost is $100 at 5 units of output, then average total cost at this output level is Multiple Choice A. $91. B. $94. Correct C. $100. D. $97.

B. $94. Correct 1. AFC = TFC/Q AFC = $100/5 = $20 2. ATC = AVC + AFC ATC = $74 + $20 = $94

The following schedule gives the cost data for a firm. Total Product Long-Run Total Cost 10 $200 20 $300 30 $450 40 $600 50 $1,000 Diseconomies of scale start between Multiple Choice A. 0 and 10 units of output. B. 40 and 50 units of output. C. 20 and 30 units of output. D. 30 and 40 units of output.

B. 40 and 50 units of output. Correct

Suppose the price of a product rises and the total revenue of sellers increases. Multiple Choice A. It can be concluded that the supply of the product is inelastic. B. No conclusion can be reached with respect to the elasticity of supply. C. It can be concluded that the supply of the product is elastic. D. It can be concluded that the demand for the product is elastic.

B. No conclusion can be reached with respect to the elasticity of supply. Correct

Suppose the price elasticity coefficients of demand are 1.43, 0.67, 1.11, and 0.29 for products W, X, Y, and Z, respectively. A 1 percent decrease in price will increase total revenue in the cases of Multiple Choice A. Y and Z. B. W and Y. C. X and Z. D. Z and W.

B. W and Y. Correct

Product reviews help to alleviate problems associated with Multiple Choice A. moral hazard. B. asymmetric information. C. negative externalities. D. positive externalities.

B. asymmetric information. Correct

If you operated a small bakery, which of the following would be a variable cost in the short run? Multiple Choice A. annual lease payment for use of the building B. baking supplies (flour, salt, etc.) C. interest on business loans D. baking ovens

B. baking supplies (flour, salt, etc.) Correct

Near an ocean beach, a high-rise building is being constructed that will block the scenic view of the ocean for the residents of a low-rise building. The Coase theorem suggests that this type of dispute between the owners of high-rise and low-rise buildings Multiple Choice A. should be resolved by a government fine for the builder of the high-rise. B. can be resolved by the owners themselves through private bargaining. C. has to be resolved by city government officials. D. can only be resolved by a zoning ordinance restricting high-rise buildings.

B. can be resolved by the owners themselves through private bargaining. Correct

Cost-benefit analysis attempts to Multiple Choice A. compare the real worth, rather than the market values, of various goods and services. B. compare the benefits and costs associated with any economic project or activity. C. determine whether it is better to cut government expenditures or reduce taxes. D. compare the relative desirability of alternative distributions of income.

B. compare the benefits and costs associated with any economic project or activity. Correct

Refer to the diagram. If actual production and consumption occur at Q2, Multiple Choice A. an efficiency loss (or deadweight loss) of e + f occurs. B. efficiency is achieved. C. an efficiency loss (or deadweight loss) of a + b + c + d occurs. D. an efficiency loss (or deadweight loss) of a + c occurs.

B. efficiency is achieved. Correct

Price per Ticket Quantity Demanded $13 1,000 $11 2,000 $9 3,000 $7 4,000 $5 5,000 $3 6,000 Refer to the information. Over the $9-$7 price range, demand is Multiple Choice A. perfectly elastic. B. elastic. C. perfectly inelastic. D. inelastic.

B. elastic. Correct

Economic profits are Multiple Choice A. equal to the difference between total revenues and implicit costs. B. equal to the difference between accounting profits and implicit costs. C. always larger than accounting profits. D. the sum of accounting profits and implicit costs.

B. equal to the difference between accounting profits and implicit costs. Correct

Refer to the provided supply and demand graph. S1 and D1 represent the current market supply and demand, respectively. S2 and D2 represent the socially optimal supply and demand. The positions of the graphs indicate that there are Multiple Choice A. external costs from production and consumption of the product. B. external benefits from production and consumption of the product. C. external benefits from production and external costs from consumption of the product. D. external costs from production and external benefits from consumption of the product.

B. external benefits from production and consumption of the product. Correct

Which of the following types of firms are least likely to have their MC, AVC, and ATC curves affected by fluctuations in gasoline prices? Multiple Choice A. companies that operate bus tours to popular vacation destinations B. firms like iTunes that distribute their products over the Internet C. firms like UPS that use a fleet of gasoline-powered vehicles D. taxi cab companies and Uber drivers

B. firms like iTunes that distribute their products over the Internet Correct

Most demand curves are relatively elastic in the upper-left portion because the original price Multiple Choice A. from which the percentage price change is calculated is small and the original quantity from which the percentage change in quantity is calculated is large. B. from which the percentage price change is calculated is large and the original quantity from which the percentage change in quantity is calculated is small. C. and quantity from which the percentage changes in price and quantity are calculated are both large. D. and quantity from which the percentage changes in price and quantity are calculated are both small.

B. from which the percentage price change is calculated is large and the original quantity from which the percentage change in quantity is calculated is small. Correct

To economists, the main difference between the short run and the long run is that Multiple Choice A. fixed costs are more important to decision making in the long run than they are in the short run. B. in the long run all resources are variable, while in the short run at least one resource is fixed. C. in the short run all resources are fixed, while in the long run all resources are variable. D. the law of diminishing returns applies in the long run, but not in the short run.

B. in the long run all resources are variable, while in the short run at least one resource is fixed. Correct

The ABC Corporation decreases all of its inputs by 12 percent and finds that its output falls by only 8 percent. This means that initially it was producing Multiple Choice A. at the point of minimum efficient scale. B. in the range of diseconomies of scale. C. in the range of economies of scale. D. where AP is less than MP.

B. in the range of diseconomies of scale. Correct

Suppose that the price of product X rises by 20 percent and the quantity supplied of X increases by 15 percent. The coefficient of price elasticity of supply for good X is Multiple Choice A. more than 1, and therefore supply is elastic. B. less than 1, and therefore supply is inelastic. C. negative, and therefore X is an inferior good. D. positive, and therefore X is a normal good.

B. less than 1, and therefore supply is inelastic. Correct

Refer to the diagram. At output level Q, Multiple Choice A. one cannot determine whether marginal product is falling or rising. B. marginal product is falling. C. marginal product is rising. D. marginal product is negative.

B. marginal product is falling. Correct

Refer to the diagrams for two separate product markets. Assume that society's optimal level of output in each market is Q0 and that government purposely shifts the market supply curve from S to S1 in diagram (a) on the left and from S to S2 in diagram (b) on the right. We can conclude that the government is correcting for Multiple Choice A. positive externalities in both diagrams. B. negative externalities in diagram (a) and positive externalities in diagram (b). C. negative externalities in both diagrams. D. positive externalities in diagram (a) and negative externalities in diagram (b).

B. negative externalities in diagram (a) and positive externalities in diagram (b). Correct

The supply curve of a one-of-a-kind original painting is Multiple Choice A. relatively inelastic. B. perfectly inelastic. C. perfectly elastic. D. relatively elastic.

B. perfectly inelastic. Correct

Suppose that the Anytown city government asks private citizens to donate money to support the town's annual holiday lighting display. Assuming that the citizens of Anytown enjoy the lighting display, the request for donations suggests that Multiple Choice A. the display creates negative externalities. B. resources are currently underallocated to the provision of holiday lighting in Anytown. C. government should tax the producers of holiday lighting. D. resources are currently overallocated to the provision of holiday lighting in Anytown.

B. resources are currently underallocated to the provision of holiday lighting in Anytown. Correct

A study of mass-transit systems in American cities revealed that in the long run, revenues generally decline after substantial fare increases. This would suggest that Multiple Choice A. the demand for mass transit is price-inelastic in the long run. B. the demand for mass transit is price-elastic in the long run. C. mass-transit service deteriorates in the long run as price rises. D. there are few good substitutes for such systems in urban areas.

B. the demand for mass transit is price-elastic in the long run. Correct

Refer to the diagrams. In which case would the coefficient of cross elasticity of demand be negative? Multiple Choice D C B A

C Correct

Refer to the provided graph showing the marginal product (MPL) and the average product of labor (APL). At which quantity of labor employed is marginal product equal to average product? Multiple Choice D A C B

C Correct

Harvey quit his job at State University, where he earned $45,000 a year. He figures his entrepreneurial talent or forgone entrepreneurial income to be $5,000 a year. To start the business, he cashed in $100,000 in bonds that earned 10 percent interest annually to buy a software company, Extreme Gaming. In the first year, the firm sold 11,000 units of software at $75 for each unit. Of the $75 per unit, $55 goes for the costs of production, packaging, marketing, employee wages and benefits, and rent on a building. The economic profits of Harvey's firm in the first year were Multiple Choice A) $70,000. B) $220,000. C) $160,000. D) $280,000.

C) $160,000. Correct Economic Profits: 100,000 * .10 = 10,000 10,000+5,000+45,000 = $160,000

Answer the question on the basis of the following information. TFC = Total Fixed Cost Q = Quantity of Output MC = Marginal Cost P = Product Price TVC = Total Variable Cost Marginal cost is _______. Multiple Choice A) (P - Q)/(change in Q) B) (Change in TFC)/(Change in Q) C) (Change in TVC)/(Change in Q) D) (Change in TVC)/(Q)

C) (Change in TVC)/(Change in Q) Correct

A firm doubles the quantity of all resources it employs and, as a result, output doubles. Which of the following is correct? Multiple Choice A) There are increasing returns to scale. B) The law of diminishing returns is proven wrong. C) The long-run average total cost curve is flat. D) The example is for the short-run rather than the long-run.

C) The long-run average total cost curve is flat. Correct

The law of diminishing returns describes the Multiple Choice A) relationship between resource inputs and product outputs in the long run. B) profit-maximizing position of a firm. C) relationship between resource inputs and product outputs in the short run. D) relationship between total costs and total revenues

C) relationship between resource inputs and product outputs in the short run. Correct

Answer the question on the basis of the following information. Number of Workers Total Product Marginal Product 0 0 --- 1 8 8 2 10 3 25 4 30 5 3 6 34 When two workers are employed, Multiple Choice A) total product is 20. B) total product cannot be determined from the information given. C) total product is 18. D) average product is 10.

C) total product is 18. Correct

Refer to the provided graph of a competitive market. If the output level is Q1, then the sum of the consumer and producer surplus is Multiple Choice A. bce. B. 0eQ1. C. 0abe. D. ac0.

C. 0abe. Correct

The supply of product X is elastic if the price of X rises by Multiple Choice A. 8 percent and quantity supplied rises by 8 percent. B. 10 percent and quantity supplied remains the same. C. 5 percent and quantity supplied rises by 7 percent. D. 7 percent and quantity supplied rises by 5 percent.

C. 5 percent and quantity supplied rises by 7 percent. Correct

Answer the question on the basis of the following demand schedule. Price Quantity Demanded $6 1 $5 2 $4 3 $3 4 $2 5 $1 6 Which of the following is correct? Multiple Choice A. Although the slope of the demand curve is constant, price elasticity increases as we move from high to low price ranges. B. Although the demand curve is convex to the origin, price elasticity of demand is constant throughout. C. Although the slope of the demand curve is constant, price elasticity declines as we move from high to low price ranges. D. A steep slope means demand is inelastic; a flat slope means demand is elastic.

C. Although the slope of the demand curve is constant, price elasticity declines as we move from high to low price ranges. Correct

Refer to the diagram. With MB1 and MC1, society's optimal amount of pollution abatement is Multiple Choice A. Q3. B. Q4. C. Q1. D. Q2.

C. Q1. Correct

(Last Word) Suppose that a firm has "pricing power" and can segregate its market into two distinct groups based on differences in elasticities of demand. The firm might charge Multiple Choice A. a lower price to the group that has the less elastic demand. B. the same price to both groups but include a "free" related product for the group that has an inelastic demand. C. a higher price to the group that has the less elastic demand. D. the same price to both groups but make it difficult for the group with the more elastic demand to gain access to the product.

C. a higher price to the group that has the less elastic demand. Correct

In the provided graph, the equilibrium point in the market is where the S and D curves intersect. At equilibrium, the producer surplus would be represented by the area Multiple Choice A. b + c. B. a + b. C. b. Correct D. b + c + d.

C. b. Correct

The demand for a product is inelastic with respect to price if Multiple Choice A. the elasticity coefficient is greater than 1. B. a drop in price is accompanied by an increase in the quantity demanded. C. consumers are largely unresponsive to a per unit price change. D. a drop in price is accompanied by a decrease in the quantity demanded.

C. consumers are largely unresponsive to a per unit price change. Correct

People enjoy outdoor holiday lighting displays and would be willing to pay to see these displays but can't be made to pay. Because those who put up lights are unable to charge others to view them, they don't put up as many lights as people would like. This is an example of a Multiple Choice A. negative externality. B. supply-side market failure. C. demand-side market failure. D. government failure.

C. demand-side market failure. Correct

The total-revenue test for elasticity Multiple Choice A. is equally applicable to both demand and supply. B. applies to the short-run supply curve but not to the long-run supply curve. C. does not apply to supply, because price and total revenue have a positive correlation. D. does not apply to demand, because price and quantity are inversely related.

C. does not apply to supply, because price and total revenue have a positive correlation. Correct

A producer's minimum acceptable price for a particular unit of a good Multiple Choice A. is the same for all units of the good. B. must cover the wages, rent, and interest payments necessary to produce the good but need not include profit. C. equals the marginal cost of producing that particular unit. D. will, for most units produced, equal the maximum that consumers are willing to pay for the good.

C. equals the marginal cost of producing that particular unit. Correct

Answer the question on the basis of the following demand schedule. Price Quantity Demanded $6 1 $5 2 $4 3 $3 4 $2 5 $1 6 The price elasticity of demand is relatively inelastic Multiple Choice A. in the $6-$4 price range. B. over the entire $6-$1 price range. C. in the $3-$1 price range. D. in the $6-$5 price range only.

C. in the $3-$1 price range. Correct

Refer to the diagram. The decline in price from P1 to P2 will Multiple Choice A. decrease total revenue by A. B. increase total revenue by B + D. C. increase total revenue by D − A. D. increase total revenue by D.

C. increase total revenue by D − A. Correct

Price per Ticket Quantity Demanded $13 1,000 $11 2,000 $9 3,000 $7 4,000 $5 5,000 $3 6,000 Refer to the information and assume the stadium capacity is 5,000. The supply of seats for the game Multiple Choice A. varies directly with ticket prices. B. is perfectly elastic. C. is perfectly inelastic. Correct D. varies inversely with ticket prices.

C. is perfectly inelastic. Correct

It is the custom for paper mills located alongside the Layzee River to discharge waste products into the river. As a result, operators of hydroelectric power-generating plants downstream along the river find that they must clean up the river's water before it flows through their equipment. In the situation described above, we would expect an Multiple Choice A. external cost resulting from the production of hydroelectric power. B. underproduction of paper in the mills. C. overproduction of paper in the mills. D. overproduction of power by the hydroelectric plants.

C. overproduction of paper in the mills. Correct

An increase in the price of tickets to a popular sporting event will increase total revenue if Multiple Choice A. there are many substitutes for this form of entertainment. B. the ticket is considered to be a luxury. C. the buyers of the tickets are fanatic about the event. D. the fans are price conscious.

C. the buyers of the tickets are fanatic about the event. Correct

Economic cost can best be defined as Multiple Choice A. any contractual obligation that results in a flow of money expenditures from an enterprise to resource suppliers. B. the opportunity cost of using a resource already owned by the firm. C. the income the firm must provide to resource suppliers to attract resources from alternative uses. D. those payments for resources that involve an obvious cash transaction.

C. the income the firm must provide to resource suppliers to attract resources from alternative uses. Correct

A negative income elasticity of demand coefficient indicates that Multiple Choice A. the product follows the law of demand. B. the product is a substitute good. C. the product is an inferior good. D. the product is a complementary good.

C. the product is an inferior good. Correct

Which of the following constitutes an implicit cost to the Johnston Manufacturing Company? Multiple Choice A) rent paid for the use of equipment owned by the Schultz Machinery Company. B) payments of wages to its office workers. C) use of savings to pay operating expenses instead of generating interest income. D) economic profits resulting from current production.

C. use of savings to pay operating expenses instead of generating interest income. Correct

If economic profits in an industry are zero and implicit costs are greater than zero, then Multiple Choice A) new firms will enter the industry. B) resources will move out of the industry. C) there will be no production in the short run. D) accounting profits are greater than zero.

D) accounting profits are greater than zero. Correct

The basic difference between the short run and the long run is that Multiple Choice A) the law of diminishing returns applies in the long run but not in the short run. B) economies of scale may be present in the short run but not in the long run. C) all costs are fixed in the short run, but all costs are variable in the long run. D) at least one resource is fixed in the short run, while all resources are variable in the long run.

D) at least one resource is fixed in the short run, while all resources are variable in the long run. Correct

At the Amarillo Piano Company, the average product of labor stays constant at 5, regardless of how much labor is employed. This implies that Multiple Choice A) this firm can never maximize its profits. B) there are no fixed costs. C) labor exhibits diminishing marginal returns. D) the marginal product of labor is constant.

D) the marginal product of labor is constant. Correct

The diagram shows the short-run average total cost curves for five different plant sizes of a firm. If in the long run the firm should produce output 0x, it should do it with a plant of size Multiple Choice A. #4. B. #3. C. #1. D. #2.

D. #2. Correct

The minimum acceptable price for a product that producer Sam is willing to receive is $15. The price he could get for the product in the market is $18. How much is Sam's producer surplus? Multiple Choice A. $270 B. $45 C. $33 D. $3

D. $3

The income elasticity of demand for food is roughly 1. A consumer's monthly income is $2,000, of which 20 percent is spent on food. If the income of this consumer doubles, the amount she'll spend on food will be Multiple Choice A. $1,000 per month. B. $400 per month. C. $500 per month. D. $800 per month.

D. $800 per month. Correct

Refer to the diagram. At output level Q, total variable cost is Multiple Choice A. BCDE. B. 0AFQ. C. 0CDQ. D. 0BEQ.

D. 0BEQ. Correct

Product % Change in Income % Change in Quantity Demanded W -1 -1 X +6 +3 Y -1 +1 Z +4 +8 Refer to the above table. Which product is a normal good but least responsive to a change in income? Multiple Choice A. Product W B. Product Z C. Product Y D. Product X

D. Product X Correct

Answer the question on the basis of the following information. TFC = Total Fixed Cost Q = Quantity of Output MC = Marginal Cost P = Product Price TVC = Total Variable Cost Average fixed cost is _______. Multiple Choice A. TVC−MC. B. MC/Q C. TVC/Q D. TFC/Q

D. TFC/Q Correct

The law of diminishing returns indicates that Multiple Choice A. the demand for goods produced by purely competitive industries is downsloping. B. because of economies and diseconomies of scale, a competitive firm's long-run average total cost curve will be U-shaped. C. beyond some point, the extra utility derived from additional units of a product will yield the consumer smaller and smaller extra amounts of satisfaction. D. as extra units of a variable resource are added to a fixed resource, marginal product will decline beyond some point.

D. as extra units of a variable resource are added to a fixed resource, marginal product will decline beyond some point. Correct

The price elasticity of demand increases with the length of the period considered because Multiple Choice A. the demand curve will shift outward as time passes. B. all prices will increase over time. C. consumers' incomes will increase over time. D. consumers will be better able to find substitutes.

D. consumers will be better able to find substitutes. Correct

For which one of the following goods would we need to sum individual demand curves vertically to obtain the total demand curve? Multiple Choice A. microwave popcorn B. frozen yogurt C. bubble gum D. courts of law

D. courts of law. Correct

A 4 percent reduction in the price of a product has zero effect on the dollar amount of consumer expenditure on the product. The price elasticity of demand is Multiple Choice A. greater than zero but less than 1. B. zero. C. greater than zero. D. equal to 1.

D. equal to 1. Correct

It is argued that, with a rising demand for college education, if the supply were to become more elastic, then college tuition costs would Multiple Choice A. decrease slowly. B. decrease steeply. C. rise faster. D. increase more slowly.

D. increase more slowly. Correct

You are the sales manager for a software company and have been informed that the price elasticity of demand for your most popular software is less than 1. In order to increase total revenues from that product, you should Multiple Choice A. decrease the price of the software. B. hold the price of the software constant. C. increase the supply of the software. D. increase the price of the software.

D. increase the price of the software. Correct

When a firm is experiencing diseconomies of scale, Multiple Choice A. it should increase the size of its plant to decrease its average total costs. B. it should lower its price to the competitive level. C. it should increase the amount of labor it hires. D. its average total costs will decline if it reduces its scale of operations.

D. its average total costs will decline if it reduces its scale of operations. Correct

The franchising of fast-food restaurants would be an example of how a private business Multiple Choice A. corrects the problem of externalities. B. expands the limits of the Coase theorem. C. solves the moral hazard problem in insurance. D. overcomes market information problems.

D. overcomes market information problems. Correct

A firm can sell as much as it wants at a constant price. Demand is thus Multiple Choice A. relatively inelastic. B. perfectly inelastic. C. relatively elastic. D. perfectly elastic.

D. perfectly elastic. Correct

Assume that a 4 percent increase in income across the economy produces an 8 percent increase in the quantity demanded of good X. The coefficient of income elasticity of demand is Multiple Choice A. negative, and therefore X is a normal good. B. positive, and therefore X is an inferior good. C. negative, and therefore X is an inferior good. D. positive, and therefore X is a normal good.

D. positive, and therefore X is a normal good.

Studies show that the demand for gasoline is Multiple Choice A. price elastic in the short run but inelastic in the long run. B. price elastic in both the short and long run. C. price inelastic in the short run but elastic in the long run. D. price inelastic in both the short and long run.

D. price inelastic in both the short and long run. Correct

A positive externality or spillover benefit occurs when Multiple Choice A. firms earn positive economic profits. B. a firm does not bear all of the costs of producing a good or service. C. product differentiation increases the variety of products available to consumers. D. the benefits associated with a product exceed those accruing to people who consume it.

D. the benefits associated with a product exceed those accruing to people who consume it. Correct

From an economist's perspective, an important consideration for policies to address global warming is Multiple Choice A. a lawsuit that can arise from the enactment of the policies. B. the supply and demand for recycled products. C. the market for recyclable inputs. D. the marginal cost and marginal benefit of the policies.

D. the marginal cost and marginal benefit of the policies. Correct

Consumer surplus arises in a market because Multiple Choice A. the market price is higher than what some consumers are willing to pay for the product. B. the quantity supplied is greater than quantity demanded at the current market price. C. the quantity demanded is greater than quantity supplied at the current market price. D. the market price is below what some consumers are willing to pay for the product.

D. the market price is below what some consumers are willing to pay for the product. Correct

Farmers often find that large bumper crops are associated with declines in their gross incomes. This suggests that Multiple Choice A. farm products are normal goods. B. the price elasticity of demand for farm products is greater than 1. C. farm products are inferior goods. D. the price elasticity of demand for farm products is less than 1.

D. the price elasticity of demand for farm products is less than 1. Correct

If there are external benefits associated with the consumption of a good or service, Multiple Choice A. the private demand curve will overestimate the true demand curve. B. the market demand curve will be the vertical summation of the individual demand curves. C. consumers are paying for all these benefits in private markets. D. the private demand curve will underestimate the true demand curve.

D. the private demand curve will underestimate the true demand curve. Correct

Suppose that Mick and Cher are the only two members of society and are willing to pay $10 and $8, respectively, for the third unit of a public good. Also, assume that the marginal cost of the third unit is $17. We can conclude that Multiple Choice A. the third unit should not be produced. B. 4 units should be produced. C. zero units should be produced. D. the third unit should be produced.

D. the third unit should be produced. Correct

Because in any period of time and in any region the quantity of pollutants that can be absorbed by nature is fixed, the supply of "pollution rights" in a cap-and-trade system will be perfectly elastic (horizontal). True or False

False

Demand-side market failures refer to those situations when there is a shortage in the market because buyers want to buy more than what is available in the market. True or False

False

Generally speaking, the smaller the percentage of one's total budget devoted to a particular product, the more price elastic will be the demand for that product. True or False

False

If the percentage change in quantity demanded is less than the percentage change in price, then demand is said to be elastic. True or False

False

If the price-elasticity coefficient for a product is 0.68 and the seller wants to raise revenues by changing its price, then the seller should cut the price of the product. True or False

False

In a well-functioning cap-and-trade system for pollution rights, society benefits because pollution will be brought down to insignificant levels. True or False

False

When a firm increases its output, its average fixed costs will stay constant. True or False

False

When diminishing marginal returns starts occurring, the addition of successive units of a variable resource to a fixed resource will cause the firm's production to diminish. True or False

False

Which of the following is not a source of economies of scale? Multiple Choice A) inelastic resource supply curves B) labor specialization C) use of larger machines D) learning-by-doing

Inelastic resource supply curves. Correct

A major factor explaining economies of scale is increased specialization of labor. True or False

True

Better Business Bureaus in various cities exist partly in order to try to deal with inadequate buyer information about sellers. True or False

True

Diseconomies of scale stem primarily from the difficulties in managing and coordinating a large-scale business enterprise. True or False

True

If changes in demand cause significant changes in equilibrium price, then supply must be quite inelastic. True or False

True

Insurance co-pays and deductibles are methods used by insurance companies to reduce moral hazard. True or False

True

Price elasticity of demand tends to be low for goods with few close substitutes. True or False

True


Set pelajaran terkait

constitution and political system- part 3

View Set

Nursing Professional Development Certification Review

View Set

Ch.12 | Domestic Violence and Other Family Health Issues Quiz

View Set

Cook's Exploration of the Pacific

View Set

Alternative AP Literature Cardset with Common Examples

View Set

Washington Heads the New Government

View Set

Science Concepts Applied to Ride

View Set

Chap 8: Government Intervention in International Business

View Set